torts BAR

Pataasin ang iyong marka sa homework at exams ngayon gamit ang Quizwiz!

when will a domestic animal subject its owner to strict liability?

- if owner knew or had reason to know of the propensity or tendency to be dangerous or agressive - the dangerous propensity is uncommon for the species - the harm foreseeably arose from the animal's uncommon dangerous propensity

the public disclosure of private facts

To recover under a theory of the public disclosure of private facts about someone, the plaintiff must show that (i) the defendant gave publicity to a matter concerning the private life of another, and (ii) the matter publicized is of a kind that would be highly offensive to a reasonable person and is not of legitimate concern to the public.

what is the implication of pure several liability?

Under a system of pure several liability, a tortfeasor is generally only liable for his comparative share of the plaintiff's damages. differs from joint and several liability, when two or more persons are responsible for a plaintiff's harm, the plaintiff may sue any one of them and obtain a full judgment.

to what torts does the doctrine of transfer intent apply? 5

1 assault 2 battery 3 false imprisionment 4 trespass to land 5 trespass to chattels

2 forms of intent

1 with the purpose of causing the result 2 with substantial certainty that the act will cause a particular result

intentional infliction of emotional distress

a defendant 1) intentionally or recklessly 2) engages in extreme and outrageous conduct 3) that causes the plaintiff severe emotional distress.

unreasonable intrusion upon the plaintiff's private affairs

the defendant's act of intruding, physically or otherwise, into the plaintiff's private affairs must be highly offensive to a reasonable person

in a trespass based on necessity, what types of damage can the property owner recover?

only actual damages. No punitive nor nominal damages are allowed.

what's the cardozo view of the scope of a defendant's general duty to freseeable victims?

a plaintiff may recover only if the P was in the zone of foreseeable danger that D's conduct created

WHAT HAPPENS Under a modified comparative negligence regime?

a plaintiff's damages are reduced by the percentage that the plaintiff is at fault.

elements of battery 3

1 intent 2 to cause 3 harmful or offensive physical contact -> not necessary to intent the contact to be harmful or offensive, only intent to cause contact

3 situations that strict liability applies

1) abnormally dangerous activities 2) wild animals and livestock 3) Manufacturing defects within products liability

elements of a prima facie of negligence claim 6

1) duty 2) standard of care 3) breach 4) actual causation 5) proximate causation 6) harm

Physicians duites?

Physicians are under a special obligation to explain all material risks of a medical procedure to a patient in advance of a patient's decision to consent to treatment. Failure of a physician to secure informed consent from the patient constitutes a breach of the physician's duty toward the patient and is actionable as medical malpractice. Doctors are not under an obligation to disclose when the risk is a commonly known risk, the patient waives or refuses the information, the patient is incompetent (although the physician must make a reasonable attempt to secure informed consent from a guardian), or disclosure would be detrimental to the patient (e.g., it would upset the patient enough to cause extreme illness, such as a heart attack).

doctrine of joint and several liability

doctrine of joint and several liability (which you should assume applies on the bar exam, unless you are told otherwise), each of two or more tortfeasors who is found liable for a single and indivisible harm to the plaintiff is subject to liability to the plaintiff for the entire harm. The plaintiff has the choice of collecting the entire judgment from one defendant, the entire judgment from another defendant, or portions of the judgment from various defendants, as long as the plaintiff's entire recovery does not exceed the amount of the judgment. Under a system of pure comparative negligence, a plaintiff's damages are reduced by the proportion that his fault bears to the total harm.

what animals are considered wild for strict liability?

- species not generally domesticated - that are likely to cause personal injury unless controlled - considered the general characteristics of the animal's species, not the particular animal involved in the legal claim

negligent infliction of emotional distress (NIED)

A plaintiff can recover for negligent infliction of emotional distress (NIED) from a defendant whose tortious conduct placed the plaintiff in harm's way if the plaintiff can demonstrate that she was within the zone of danger and the threat of physical impact caused emotional distress. Generally, the distress must exhibit some physical symptoms. In virtually all jurisdictions, emotional distress must result from sensory and contemporaneous observance of the accident itself, not the receipt of news relating to the accident. To recover as a bystander, the plaintiff must be closely related to a person injured by the defendant, be present at the scene of the injury, and personally observe or perceive the injury.

Plaintiff and bystander for NIED?

A plaintiff can recover for negligent infliction of emotional distress from a defendant whose tortious conduct placed the plaintiff in harm's way if the plaintiff demonstrates that: (i) he was within the "zone of danger" of the threatened physical impact—that he feared for his own safety because of the defendant's negligence; and (ii) the threat of physical impact caused emotional distress. However, a bystander plaintiff who is outside the zone of danger can still recover if she is closely related to the person injured by the defendant, she was present at the scene of the injury, and she personally observed (or otherwise perceived) it.

elements of defamation

A plaintiff may bring an action for defamation if the defendant's defamatory language of or concerning the plaintiff is published to a third party who understands its defamatory nature, and it damages the plaintiff's reputation.

what is the actual malice requirement for defamation of public figures>

Actual malice can be established by showing that the defendant either had knowledge that the statement was false or acted with reckless disregard as to the truth or falsity of the statement. To establish a reckless disregard for the truthfulness of a statement, the plaintiff must prove that the defendant entertained serious doubts about the statement's truthfulness; mere failure to check facts is not sufficient.

A woman decided to take up beekeeping as a hobby. She purchased some beehive boxes, worker bees, and one queen bee, and began making honey in her backyard bee farm. The woman's bees did often leave the beehive boxes and occasionally ventured onto neighboring properties, but there were still fewer bees than one might encounter near a typical flower bush or in a park. A man that lived next door to the woman was extremely allergic to bee stings, and if stung, would immediately have to go to the hospital. Because of his extreme allergy, the man rarely left the house. When the man saw the beehive boxes in the woman's backyard, he sued her for private nuisance.Is the man likely to prevail? A No, because the average person would not be inconvenienced by the beehive boxes. B No, because the man rarely left his home due to the presence of the bees. C Yes, because the man had an extreme allergic reaction to bee stings. D Yes, because the beehive boxes were an unreasonable interference.

Answer choice A is correct. A private nuisance is a thing or activity that substantially and unreasonably interferes with another individual's use or enjoyment of his land. A person with special sensitivities can recover only if the average person would be offended, inconvenienced, or annoyed. Here, since the beehives housed fewer bees than one might encounter near a typical flower bush or in a park, the average person would be unlikely to be inconvenienced, annoyed, or offended by the bees. Answer choice B is incorrect because even if the man rarely leaves his home, he has a possessory interest in his property. This would make him a proper plaintiff if there had also been a substantial interference that would be offensive, inconvenient, or annoying to a normal, reasonable person in the community. Answer choice C is incorrect because the man's sensitivity to bee stings does not change the standard that must be met to establish a substantial interference for a private nuisance tort. Answer choice D is incorrect. Here, the interference was not unreasonable because raising bees and making honey is a useful hobby, and outweighs the minor issue of a few bees occasionally visiting neighboring properties.

An avid runner was diagnosed with a serious heart condition. The runner's doctor advised her to avoid strenuous physical activity, including running, as such activity would create a substantial risk of cardiac arrest. The runner refrained from such activity for a month, but in that time she gained 15 pounds and felt very unhealthy. Deciding that the health benefits of running outweighed the risk involved, one morning she set out on her normal running path—the shoulder of a flat rural road. Five miles into the run, the runner suffered a heart attack, collapsed, and lapsed into a coma. Two minutes later, the runner's feet and legs—which were partially sticking out into the travel lane—were run over by a car. The driver of the car, who had been traveling at a reasonable speed, was aware of the runner but was unable to avoid her due to a locking up of the car's brakes that the driver had negligently failed to have repaired. The runner survived, but suffered serious injuries to both of her legs. The runner sued the driver for those injuries in a jurisdiction that applies traditional contributory negligence rules. Is the runner likely to prevail? A No, because the runner was contributorily negligent. B No, because the driver was aware of the runner's predicament before the accident occurred. C Yes, because of the runner's helpless peril at the time of the accident. D Yes, because the driver was negligent in driving the car with brakes in need of repair.

Answer choice A is correct. In a contributory negligence jurisdiction, the plaintiff's contributory negligence is a complete bar to the plaintiff's recovery. Here, the runner was negligent in running against her doctor's advice. Consequently, despite the driver's negligence in failing to properly maintain his car, which led directly to the accident, the runner cannot recover from the driver. Answer choice B is incorrect because, although the driver was aware of the runner's predicament before the accident, the driver could not avoid hitting the runner by acting reasonably. The driver's negligence (i.e., his failure to repair the car's brakes) occurred prior to his awareness of the runner's predicament and could not be corrected after gaining such awareness before the accident. Answer choice C is incorrect because, although the runner was unable to extricate herself from the situation since she was unconscious, the driver may be liable if he had the last clear chance to avoid the accident. However, as explained with regard to answer choice B, the driver did not have the last clear chance to avoid the accident and is not liable. Answer choice D is incorrect because, in a contributory negligence jurisdiction, the negligence of the defendant is not sufficient to permit the plaintiff to recover where the plaintiff is also negligent.

A patron at a resort ranch took part in a supervised horseback trail ride. Prior to the ride, the patron executed a valid release that enumerated the inherent risks of horseback riding and, by its terms, relieved the resort from liability from any loss, damage, or injury to the guest's person or property suffered during the ride attributable to the negligence of the ranch or its employees. The patron was injured by a fall from the horse. The horse reared in response to negligent behavior of another rider who was also a patron at the ranch. The patron filed suit against the ranch and the other rider for damages resulting from his injuries that totaled $400,000. At trial, it was determined that the ranch was 75% at fault for the patron's injuries due to its selection and training of the horse, and that the other rider was 25% at fault. The applicable jurisdiction recognizes the validity of such releases and has enacted both a modified comparative negligence statute and a pure several liability statute. How much can the patron recover from the ranch? A Nothing B $100,000 C $300,000 D $400,000

Answer choice A is correct. In general, parties can contract to disclaim liability for negligence. An exculpatory provision in a contract acts as a bar to recovery for harms arising from the negligence of the party protected by the contract. This bar applies even where the state has adopted a comparative negligence statute. Here, the patron entered into a valid agreement to exculpate the ranch from liability for its negligence. Consequently, answer choices B, C, and D are incorrect.

The owner of a lakefront home in a retirement community that greatly restricts access by nonresidents was aware that her dock needed repair, but was unable to afford the considerable expense to do so. The owner placed a large heavy chair at the entrance to the dock with a sign that read, "Please do not enter. Dock in need of repair." Two children, a six-year-old boy and a ten-year-old girl, entered the property without permission from, or knowledge of the owner. The children quickly discovered the dock. The girl read the sign aloud to the boy and advised him, "You shouldn't go out on the dock." The boy, responding "But it's not dangerous," climbed over the chair and walked out onto the dock. As the boy ran to the end of the dock, a rotten plank on which the boy stepped gave way, and he fell into the lake and drowned. As permitted by the applicable jurisdiction, the boy's parents sued the owner in a wrongful death action alleging that her negligence with respect to the dock caused the boy's death. At trial, the boy's parents argued that the dock constituted an attractive nuisance. Which of the following may protect the owner from liability that otherwise would arise under this doctrine? A The owner lives in a retirement community that greatly restricts access by nonresidents. B The boy was not attracted to the property by the presence of the dock. C The boy was a trespasser. D The boy was aware of the owner's warning.

Answer choice A is correct. In order for the attractive nuisance doctrine to apply, the landowner must know or have reason to know that the artificial condition is located in a place that children are likely to trespass. Because the owner lives in a retirement community that greatly restricts access by nonresidents, this requirement is not satisfied. Answer choice B is incorrect because, despite its name, the attractive nuisance doctrine does not require that the child be enticed onto the property by the presence of the condition. Answer choice C is incorrect because the purpose of the attractive nuisance doctrine is to permit a child trespasser to recover for a landowner's negligence. Answer choice D is incorrect because in order for the attractive nuisance doctrine to apply, the child, due to his youth, must not appreciate the danger presented by the condition. A warning will often protect a landowner from the liability by bringing the condition to the attention of the child. In this case, despite the warning, the child, while deciding that he should go on the dock, did not realize that the reason he should not do so was its dangerous condition.

A patient visited his physician to get an influenza vaccine. Before seeing his physician, the patient completed a form that, among other things, asked the patient to list his allergies. The patient, believing that the question referenced allergies to medication, listed the medications to which he is allergic. He did not list his food allergies, which include eggs. Before administering the vaccine, the physician had a discussion with the patient regarding various side effects associated with the vaccine. They did not specifically discuss the patient's allergies. The physician administered the vaccine, which is made with eggs. The patient had a severe allergic reaction, and died shortly thereafter.Would the physician be liable in a negligence action for the patient's death? A Yes, because the physician did not obtain informed consent. B Yes, because the physician did not exhibit the same skill, knowledge, and care of a doctor of similar education and experience. C No, because the physician exhibited the same skill, knowledge, and care as another physician in the community. D No, because the physician acted in a reasonably prudent manner.

Answer choice A is correct. Negligence is the commission of an act (or the failure to act), without wrongful intent, that falls below the minimum degree of ordinary care imposed by law to protect others against unreasonable risk of harm. Physicians are under a specific obligation to explain the risks of a medical procedure to a patient in advance of a patient's decision to consent to treatment. Failure to comply with this "informed consent" doctrine constitutes a breach of the physician's duty owed to the patient and is actionable as medical malpractice (medical negligence). By failing to discuss the patient's allergies, and the risk of death from the eggs in the vaccine, the doctor breached his duty to the patient to obtain informed consent. Answer choice B is incorrect because it basically describes the standard of care imposed upon children rather than professionals (though substituting "education" for "age"). Answer choice C is incorrect because, although it lists the correct standard for professionals, it fails to account for a physician's duty to obtain informed consent. Answer choice D is incorrect because a physician is under a special duty to obtain informed consent, which the physician failed to do in this case.

The defendant owns an alligator, which she keeps in a pen in her front yard; local laws do not prohibit owning such an animal. The plaintiff was walking by the defendant's yard on the sidewalk when he noticed the alligator. Although the plaintiff was not in actual danger from the alligator, he jumped backwards in fear when he saw the creature, and severely sprained his ankle. The plaintiff sued the defendant to recover damages for injury.Of the following, which provides the defendant with her best argument against this claim? A The alligator was properly restrained when the plaintiff suffered his injury. B The plaintiff cannot recover because he was in no actual danger from the alligator. C No local law precludes the defendant from owning an alligator. D The plaintiff cannot recover because his injury was not caused by the alligator's dangerous propensities.

Answer choice A is correct. The possessor of a wild animal is strictly liable for harm done by that animal, in spite of any precautions the possessor has taken to confine the animal or prevent the harm, if the harm arises from a dangerous propensity that is characteristic of such a wild animal or of which the owner has reason to know. Strict liability also applies to an injury caused by a plaintiff's fearful reaction to the sight of an unrestrained wild animal. In this case, although the defendant owned a wild animal, she is not strictly liable for the injury caused by the plaintiff's fearful reaction to the sight of the alligator because the alligator was not roaming free, but instead was confined within a pen. Consequently, the defendant is not liable to the plaintiff in negligence, nor is she strictly liable, because the alligator was properly confined. Answer B is incorrect because strict liability for harm caused by wild animals extends to harm caused by the reasonable fear people feel in the vicinity of the wild animal, if unrestrained. Answer choice C is incorrect because compliance with a statute, a regulation, or an ordinance generally does not prove the absence of negligence, and would not protect the defendant from strict liability if the animal had escaped. Answer choice D is incorrect because the injury need not be caused by the alligator's dangerous propensities; an injury caused by the plaintiff's fearful reaction to an unrestrained wild animal is sufficient.

A plaintiff was on a crowded subway train during rush hour. The subway line was undergoing significant renovations, resulting in frequent, sudden stops by the subway trains. The plaintiff was standing in the middle of one of the subway cars and holding onto a pole for stability. The defendant, also standing in the subway car, was texting on his cell phone and not holding onto anything. The subway train came to a sudden stop causing the defendant to fall toward the plaintiff. The defendant lightly grabbed the plaintiff's arm to stop himself from falling completely over. The plaintiff did not like being touched by anyone as she had been in an abusive relationship in the past. Although she was not injured by the defendant's conduct, the plaintiff subsequently brought an action for battery against the defendant.Will the plaintiff prevail? A No, because the plaintiff consented to the defendant's contact. B No, because the plaintiff did not suffer any actual harm. C Yes, because the defendant failed to exercise reasonable care. D Yes, because the defendant intentionally grabbed her arm.

Answer choice A is correct. There is no battery if the plaintiff consented to the act, either expressly or by virtue of participating in a particular event or situation (such as being bumped on a crowded subway). In this case, the plaintiff chose to ride a crowded subway train. By making that decision, she implicitly consented to the ordinary contacts that can occur in that situation. The defendant lightly grabbing her arm to prevent himself from falling is within the type of contact that is typical on a crowded subway train. In addition, although the plaintiff found the contact offensive, a contact is only offensive when a person of ordinary sensibilities (i.e., a reasonable person) would find the contact offensive (objective test). Answer choice B is incorrect because proof of actual harm is not required to recover for battery. Answer choice C is incorrect. While the defendant's failure to exercise reasonable care would be relevant to a negligence claim, it is not relevant to a claim for battery. Answer choice D is incorrect. The fact that the defendant intentionally grabbed the plaintiff's arm to prevent his fall is not enough to impose liability for battery. Here, the contact was neither harmful nor offensive. In addition, the plaintiff impliedly consented to the contact. LEARN WHYNEXT QUESTION

A pedestrian was walking next to a building under construction. Suddenly, he was hit in the head by a falling brick. As a consequence, the pedestrian suffered a skull fracture and a severe brain injury. The pedestrian sued the construction company. At trial, the pedestrian did not introduce any direct evidence of the construction company's negligence, but proved that the construction company's employees were in control of its bricks at all relevant times, and that a brick does not ordinarily fall from a building under construction without negligence. The construction company offered uncontroverted proof that the pedestrian was negligent by walking so close to an active construction site. The jurisdiction in which the lawsuit is proceeding applies pure comparative negligence rules. At the close of all evidence, the construction company moved for a directed verdict. Should the court deny this motion? A YES, because the pedestrian's negligence does not reduce the likelihood of the construction company being negligent. B Yes, because res ipsa loquitur requires a finding of negligence. C No, because a party who is negligent may not prevail under a res ipsa loquitur theory. D No, because the pedestrian has not produced any direct evidence of the company's culpability.

Answer choice A is correct. To obtain a res ipsa loquitur jury instruction, a plaintiff must prove that (i) his injury was caused by an instrumentality or agent within the exclusive control of the defendant, (ii) the accident was of a kind that ordinarily does not occur in the absence of negligence, and (iii) the harm was not due to any action on the part of the plaintiff. The third requirement is not satisfied if a plaintiff's own negligence increases the likelihood of the defendant's negligence. In this case, the pedestrian's negligence (his proximity to the construction site) had nothing to do with the construction company's negligent control of the brick. Answer choice B is incorrect because a res ipsa loquitur instruction permits the jury to find negligence even in the absence of direct evidence, but does not require the jury to do so. Answer choice C is incorrect because it is an inaccurate statement of the law; a plaintiff's own negligence does not necessarily prevent him from recovering under res ipsa loquitur. Answer choice D is incorrect because res ipsa loquitur allows a finding of negligence even in the absence of direct evidence. Note: Courts in the vast majority of jurisdictions that have adopted comparative fault (such as in this case) also are inclined to loosely apply the third requirement—that the harm must not be due to any action on the part of the plaintiff—because such a requirement would otherwise be in tension with the law holding that the plaintiff's contributory negligence is no longer a total bar to recovery.

A woman placed an online order to purchase a unique tea kettle from a kettle manufacturer as a gift for her brother. The kettle was square-shaped and had two spouts. The kettle's shipping box stated the following: "Caution! The enclosed kettle is a novelty item and should not be used to pour boiling water. Use with boiling water may result in steam burns." Before using the kettle, the brother read some online reviews of the kettle. Many reliable reviews stated that steam burns were very common when using the kettle, because steam would escape from the second spout if the kettle was not held properly. However, many reviews suggested that the kettle could be used safely if it was held in a particular way. The first time the brother used the kettle, he carefully followed the instructions from the reviews on how to hold the kettle when filled with boiling water. However, while he was pouring the boiling water from one spout, scorching steam escaped from the other spout and burned the brother's skin. The jurisdiction applies the common-law rules for contributory negligence and assumption of the risk.If the brother files a strict-products-liability suit against the manufacturer, what is the manufacturer's best defense? A The brother assumed the risk of being burned. B The brother did not use the kettle properly. C The brother received the kettle as a gift. D The brother was negligent in using the kettle with boiling water.

Answer choice A is correct. Under the doctrine of strict products liability, a seller of a product is liable for personal injuries caused by that product, even in the absence of fault, if the product was defective, the defect existed at the time the product left the defendant's control, and the defect caused the plaintiff's injury when the product was used in an intended or reasonably foreseeable way. However, a voluntary and knowing assumption of the risk is a complete bar to recovery in contributory-negligence jurisdictions if a plaintiff is aware of the danger and knowingly exposes himself to it. Here, the brother knew that steam burns were possible if the novelty kettle was used with boiling water, but decided to use the kettle with boiling water anyway. Accordingly, assumption of the risk provides the manufacturer with the strongest defense to the brother's strict-products-liability suit. Answer choice B is incorrect. Mere misuse will not constitute a defense to a strict products liability claim if the misuse is reasonably foreseeable. Here, the fact that the brother misused the kettle would likely be a foreseeable misuse of the kettle, especially based on the number of online reviews indicating users had used the kettle with boiling water. Therefore, this argument would not protect the manufacturer from liability. Answer choice C is incorrect. Anyone foreseeably injured by a defective product may bring a strict-liability action. Appropriate plaintiffs include not only purchasers, but also other users of the product. There is no requirement that the brother must be a purchaser in order to recover. Answer choice D is incorrect. In a contributory-negligence jurisdiction, the plaintiff's negligence generally is not a defense to a strict-products-liability action when the plaintiff negligently failed to discover the defect or misused the product in a reasonably foreseeable way. Ordinary contributory negligence by the plaintiff will not bar recovery based on strict products liability. Therefore, the manufacturer will only have a chance to succeed in defending against this action if it argues that the brother assumed the risk of steam burns. LEARN WHYNEXT QUESTION

An eight-year-old girl started to constantly tease and harass a classmate. One day, the classmate needed to use the restroom and tried to go into the bathroom. Knowing how much it would upset the classmate, the girl physically blocked the door, told the classmate she could not use the bathroom, and then pushed the classmate, hoping she would fall to the ground. The classmate struck her head on the edge of a desk as she fell, sustaining a serious injury.If the classmate's parents assert a claim on behalf of their daughter for damages against the girl, will they succeed? A Yes, because the girl intended to cause a serious injury. B Yes, because the girl's intentional conduct caused the classmate's injury. C No, because the girl is too young to be held liable for an intentional tort. D No, because only the girl's parents can be held liable for her tortious conduct.

Answer choice B is correct. A majority of courts hold that children can be held liable for intentional torts if they either act with a purpose or know the consequences of their acts with a substantial certainty. One is liable for battery when he intentionally engages in a harmful or offensive contact with another without the other's consent. Here, the girl intentionally pushed the classmate and the classmate was harmed as a result. Therefore, the action will succeed. Answer choice A is incorrect. In a battery action, there is no requirement that a defendant intend to cause a serious injury or even the specific harm that the plaintiff suffered. Answer choice C is incorrect because there is no minimum age requirement in order to be liable for an intentional tort. Answer choice D is incorrect because, in general, parents are not vicariously liable for the torts of their children. Although they can be directly liable for their own negligence, there is no evidence of such negligence in this fact pattern. Additionally, the classmate's parents are not suing the parents directly. LEARN WHYNEXT QUESTION

A masseur operates a massage therapy parlor out of his home in a rural neighborhood, and clients regularly enjoyed receiving massages in the masseur's custom-built outdoor therapy room. The landowner's only neighbor, whose property was adjacent to the landowner's home, recently started a non-profit rescue program for mistreated farm animals. The neighbor built a number of small enclosures in her own yard to house the rescued animals until they could be rehomed. Although the animal rescue program was properly registered and in compliance with local zoning codes, the constant smell and noise from the animals interfered with the masseur's business. Many of the masseur's regular clients who benefited from the outdoor therapy room stopped coming because the stench and noise of the livestock were too distracting. As a result, the masseur has lost significant business.Which of the following actions gives the masseur the best chance to recover any economic damages from the neighbor? A Attractive nuisance doctrine B Private nuisance C Strict liability for damages caused by animals D Trespass to land

Answer choice B is correct. A private nuisance is a thing or activity that substantially and unreasonably interferes with another's use or enjoyment of her property. In this case, the masseur could argue that the animal rescue program created a substantial interference because it created a stench and noise that was annoying and disruptive to his clients. Answer choice A is incorrect because the attractive nuisance doctrine only applies when there has been an injury to a trespassing child, which is not relevant here. Answer choice C is incorrect. Regardless of whether these farm animals are classified as wild or domestic animals, to succeed in a strict liability action based on injuries caused by animals, the injury must be a result of either a dangerous propensity of the animal, a plaintiff's fearful reaction to the sight of an unrestrained wild animal, or a trespassing animal. Because no dangerous propensity of any animal caused the landowner's economic injuries in any way, and because the animals never trespassed on the masseur's land, this cause of action is inappropriate. Answer choice D is incorrect because a trespass to land occurs when the defendant's intentional act causes a physical invasion of the land of another. A scent is not considered a physical invasion of land.

A lawyer reserved a room at a hotel the night before a deposition. A stalker who had been following the lawyer for months decided to leave a box of peanut butter chocolates in the lawyer's room before she arrived. Using a fake driver's license with the same last name as the lawyer, he went up to the front desk, pretended to be the lawyer's husband, and requested a key card to her room. The receptionist at the front desk accidentally gave the stalker a key card to a room belonging to a different woman with the same name as the lawyer. The stalker went to that room, found it empty, and left the box of chocolates on the bed with a note stating, "For the love of my life." When the woman came back to her room, she assumed that her husband had sent the chocolates. The woman was severely allergic to nuts, but because her husband was aware of her allergy, she assumed the chocolates were nut free. Shortly after eating a peanut butter chocolate, she went into anaphylactic shock and nearly died. The woman subsequently sued the receptionist to recover damages for her personal injuries. The court has concluded that the receptionist breached a duty to the woman by giving the stalker a key card to her room.On these facts, is the woman likely to recover her damages? A No, because the stalker, not the receptionist, used the negligently issued key card. B No, because the stalker's conduct and the woman's resulting allergic reaction were not foreseeable. C Yes, because the receptionist breached his duty of care to the woman. D Yes, because the receptionist is liable for the woman's harm under the "eggshell-skull" rule.

Answer choice B is correct. In order to establish a prima facie case of negligence, one must prove duty, breach, actual and proximate causation, and damages. Here, duty and breach have been established, and the woman suffered damages. Thus, the remaining issue is whether the woman's damages are causally linked to the receptionist's breach such that the receptionist is liable. Here, the receptionist's negligence is an actual cause of the woman's injury because the stalker would not have been able to leave the chocolates in the woman's room without the receptionist's negligent issuance of the key card. However, his breach is not the proximate cause of these injuries. The majority rule for proximate cause requires that the plaintiff suffer a foreseeable harm that is not too remote and is within the risk created by the defendant's conduct. The type of harm must be foreseeable, though the extent of harm need not be foreseeable. Here, the type of harm suffered by the woman is unforeseeable and outside the scope of the risk that would typically be created by the receptionist's negligence. If the woman had been attacked or robbed by someone using the negligently issued key card, that would likely be within the scope of the risk created by the receptionist's breach. However, an allergic reaction to a mistaken gift of chocolates is damage of a very different type, and is therefore too remotely connected to the receptionist's negligence for proximate cause to exist. Answer choice A is incorrect because the receptionist could still be liable if the stalker had caused an injury within the scope of risk created by the receptionist's negligence. For example, if the stalker had used the key card to enter the room and rob the woman, the receptionist's negligence would likely be an actual and proximate cause of that injury. Answer choice C is incorrect because although the receptionist breached his duty of care, the woman must also establish actual and proximate causation to recover. Answer choice D is incorrect. Although the extent of damages need not be foreseeable for the plaintiff to recover, the type of damages does need to be within the scope of risk created by the defendant's breach in order for proximate cause to exist. Because the "eggshell-skull" rule only applies to make the defendant liable for the unforeseeable extent of a foreseeable type of harm, it does not apply here. LEARN WHYNEXT QUESTION

As a safety measure, a city enacted a code imposing a fine for trespassing on construction and roadwork sites marked with orange caution tape. The city's construction team marked a city block with orange caution tape to divert all cars and pedestrians while the team repaired potholes. A pedestrian wanted to walk down the marked-off street. The street was full of construction equipment, but because it appeared that construction had not yet started, the pedestrian stepped over the orange caution tape and walked down the street. Distracted by the equipment around him, the pedestrian stepped into an open manhole and landed on a construction worker who was inspecting the sewer under the construction site. Both parties were injured in the accident. The construction worker has sued the pedestrian for negligence to recover damages for his injuries in a jurisdiction that applies the minority approach to negligence per se.What is the legal effect of the pedestrian's walking on a street marked with caution tape? A It creates a conclusive presumption that the pedestrian breached a duty. B It creates a rebuttable presumption that the pedestrian breached a duty. C It will bar the pedestrian from recovering damages from the city. D It will have no legal effect because the construction worker's act was the proximate cause of the accident.

Answer choice B is correct. In some cases, the standard of care owed by a defendant can be determined by statute. In most jurisdictions, this violation can establish negligence as a matter of law. In a minority of jurisdictions, however, violation of that statute can give rise to a rebuttable presumption that the defendant owed a duty and breached it. This violation is referred to as negligence per se. Here, the pedestrian violated a code that imposed a fine for trespassing on construction sites, which the pedestrian did. Therefore, in a minority jurisdiction such as this one, this violation creates a rebuttable presumption that the pedestrian breached a duty. Accordingly, answer choice A is incorrect. Answer choice C is incorrect. The default rule on the MBE is pure comparative negligence, unless the facts instruct you to apply a different test. In a pure comparative negligence jurisdiction, a plaintiff's contributory negligence is not a complete bar to recovery. Therefore, the fact that the pedestrian may have been negligent as a matter of law would not prevent the pedestrian from recovering damages from the city (if the city was somehow negligent). Answer choice D is incorrect. Even if the worker's act contributed to the cause of the accident, the pedestrian's violation of the code has the legal effect of creating a rebuttable presumption that the pedestrian breached a duty.

Over the course of one night, an attorney went to three different bars: Bar A, Bar B, and Bar C. The attorney stayed at each bar for roughly equal amounts of time, and each bar served him enough liquor to make him legally intoxicated. At the end of the night, the attorney left Bar C and was driving home erratically. A block away from his home, the attorney lost control of his car, careened into oncoming traffic, and collided with his neighbor's car. The attorney died in the collision, and his neighbor was permanently disfigured. The neighbor sued Bar A in a jurisdiction that has adopted standard dram shop laws. Bar A filed a motion to dismiss the suit for failing to state a claim upon which relief can be granted. How should the court rule on the motion? A Deny the motion, and order Bar B, Bar C, and the attorney's estate joined as defendants. B Deny the motion, because a reasonable fact finder could determine that the neighbor's injuries were a continuing consequence of Bar A's actions. C Grant the motion, because the attorney's criminal act of driving drunk was a superseding cause that cut off Bar A's liability. D Grant the motion, because Bar A's negligence was not the "but-for" cause of the neighbor's injuries.

Answer choice B is correct. The attorney's drunk driving was a foreseeable continuing consequence of Bar A's negligent activities. Thus, Bar A can be held liable for the accident, because its actions were a "substantial factor" in causing the neighbor's injuries. Bar A may not escape liability simply because other negligent actors exist. Answer choice A is incorrect because Bar A, as a substantial factor in causing the neighbor's injuries, can be held liable for all of his damages. Answer choice C is incorrect because the attorney's drunk driving was foreseeable, and thus was not a superseding cause. Answer choice D is incorrect because Bar A's actions were a substantial factor in causing the neighbor's injuries, and thus are considered a cause-in-fact.

Using a path frequented by students, a college student decides to take a short cut through the back yard of a homeowner. The homeowner, who is unaware of the student's presence, is cleaning out the cage of a rattlesnake he is keeping at his house. The homeowner has carelessly allowed the snake to roam free. The snake, hidden from the student's view by a tree, is startled by the student as the student walks past and strikes the student. The student is seriously injured by the snakebite.The applicable jurisdiction permits the keeping of a rattlesnake as a pet.In a strict liability action by the student against the homeowner, who will prevail? A The student, because the homeowner possessed a wild animal. B The student, because the homeowner, aware that students frequently used the path, failed to act with reasonable care. C The homeowner, because the student was trespassing. D The homeowner, because the homeowner's possession of the rattlesnake is legal.

Answer choice C is correct because an owner of a wild animal is generally not strictly liable to a trespasser who is injured by the wild animal, except for injuries caused by a vicious watchdog. Here, the student was a trespasser and the homeowner's animal was not a vicious watchdog. Answer choice A is incorrect because, while the possessor of a wild animal generally is strictly liable to a licensee or invitee who is injured by the wild animal, the possessor is not strictly liable to a trespasser. Answer choice B is incorrect because the action brought by the student was based on strict liability, not negligence. Consequently, the homeowner's failure to act with reasonable care is irrelevant. Answer choice D is incorrect because, even though the homeowner's possession of the rattlesnake is legal, the rattlesnake is a wild animal. As such, the possessor of a wild animal may be strictly liable for harm caused by the animal, unless the individual harmed is a trespasser. LEARN WHYNEXT QUESTION

In a well-trafficked downtown location, a voyeur concealed a video camera near a sidewalk grate. As the voyeur was aware, a natural spurt of air coming up from the grate would occasionally lift a woman's skirt and reveal her underwear. In reviewing the video taken one day, the voyeur discovered a short sequence involving a prominent female politician who at the time was not wearing underwear. The voyeur contacted the politician and demanded a substantial payment in exchange for not posting the video on the Internet. The politician sued the voyeur in an invasion of privacy action based on intrusion upon her seclusion. The voyeur moved to dismiss the action for failure to state a cause of action. Should the court grant this motion? A No, because the politician did not consent to the video. B No, because the video intruded into her privacy in a manner highly offensive to a reasonable person. C Yes, because the video was made in a public place. D Yes, because the video was not revealed to a third party.

Answer choice B is correct. The defendant's act of intruding, physically or otherwise, into the plaintiff's private seclusion if the intrusion is highly offensive to a reasonable person establishes liability. Here, the video pried into a matter over which the plaintiff retained a privacy interest (i.e., the existence or lack of underwear) and the manner in which the voyeur did so (i.e., by a hidden camera) was highly offensive to a reasonable person. Answer choice A is incorrect because, while a plaintiff's consent to the defendant's intrusion into a secluded area is a defense to this type of invasion of privacy action, the absence of such consent does not necessarily give rise to a cause of action. Answer choice C is incorrect because, while this type of invasion of privacy action generally does not protect a plaintiff in a public place, there are some matters, such as the existence or lack of underwear, about which a plaintiff retains an expectation of privacy even in a public place. Answer choice D is incorrect because, unlike other types of invasion of privacy actions, intrusion upon seclusion does not require that the private information about the plaintiff be published to a third party.

In a valid contract, the plaintiff promised to pay the defendant $87,000 to fumigate the plaintiff's commercial office building within seven days to stop a major insect infestation. The defendant performed the fumigation, and plaintiff paid the $87,000. Two months later, the plaintiff filed a complaint in the State A federal district court, making three main allegations. First, "Plaintiff is a State Z citizen, Defendant is a State A citizen, the amount in controversy is $87,000, and the court has diversity jurisdiction." Second, "Defendant breached its contract with Plaintiff (copy attached) by failing to render adequate performance, and Plaintiff has been unable to sell his commercial office building." Third, "Plaintiff demands judgment of $87,000, the amount Plaintiff lost as a result of Defendant's breach." What would be the defendant's best response? A Filing a Rule 12(b) motion to dismiss for lack of subject matter jurisdiction. B Filing a Rule 12(b)(6) motion to dismiss for failure to state a claim upon which relief can be granted. C Filing an answer denying the plaintiff's allegation that the defendant breached the contract. D Filing a motion for summary judgment on the grounds that there is no genuine issue of material fact and that the defendant is entitled to judgment as a matter of law.

Answer choice B is correct. The plaintiff has failed to allege facts sufficient to support a cognizable legal claim. Rather, the plaintiff has simply made the conclusory assertion that the defendant breached the contract by failing to perform adequately, without setting forth any facts explaining why the performance is inadequate under the contract. The fact that the plaintiff has been unable to sell his building is irrelevant, as this inability might reflect any number of factors unrelated to the quality of the building fumigation (such as a poor economy or an unreasonably high asking price). Answer choice A is incorrect because the facts alleged, if true, establish subject matter jurisdiction (diversity of citizenship). Answer choice C is incorrect because the defendant's best response would be to obtain a motion to dismiss, rather than to answer the claim and become mired in a lawsuit (even though an answer would be a lawful option). Answer choice D is incorrect because a summary judgment motion would be appropriate only if matters outside the pleadings had been introduced.

In a bicycle race with a $5,000 prize for the winner, a cyclist was leading by a significant margin. A spectator at the race was married to the second place rider. Sensing that her husband would not win unless she took action, the spectator drove to a point two miles ahead on the course, scattered several nails in the middle of the course, and then left the area. Soon thereafter, the cyclist approached the area and noticed the nails. He attempted to swerve around the obstruction but a nail punctured his tire. He fell off his bike, suffered significant physical injuries, and was unable to complete the race. If the cyclist sues the spectator, under what theory is the cyclist least likely to recover maximum punitive damages? A Assault B Intentional infliction of emotional distress C Trespass to chattels D Battery

Answer choice B is correct. To sustain a claim of intentional infliction of emotional distress, a plaintiff must prove that the defendant intended to cause severe emotional distress, or acted with recklessness as to the risk of causing such distress. The claimed conduct must be extreme and outrageous, and the plaintiff must suffer severe emotional distress. Here, there is no factual evidence that the bicyclist suffered severe emotional distress. A plaintiff can recover punitive damages in assault, battery, and trespass to chattels claims, if the defendant's behavior is willful and wanton, reckless, or if the defendant acted with malice. Here, the facts suggest such a situation, as the spectator was attempting to knock the cyclist out of the race. For these reasons, answers A, C, and D are incorrect.

In reporting on the death of a city official whose bullet-ridden body was found in a barren apartment, a newspaper attributed the death to a "drug deal that went sour." The newspaper reporter who filed the report had serious doubts about the official's involvement with drugs. Later, the newspaper determined that the official neither used nor sold illegal drugs, but instead was killed because he had been involved in a fraud scheme that went awry. The executor of the official's estate brought an action for defamation against the newspaper. The executor is unable to establish special damages. Who will prevail? A The executor, because presumed damages are permitted for a libel action. B The executor, because the newspaper acted with malice. C The newspaper, because the city official was dead. D The newspaper, because the statement regarding the city official's involvement in criminal activity was substantially true.

Answer choice C is correct. A deceased person cannot legally be defamed. The estate of the deceased official cannot maintain an action for defamation because the defamatory statement was made after the official's death. Answer choice A is incorrect, because, even though presumed damages are permitted in a libel action, there is no basis for an action under these facts. Answer choice B is incorrect because, even though the newspaper acted with a reckless disregard for the truth of its statement about the official's involvement in illegal drug activity, which is sufficient to constitute malice, the statement was made about a deceased person. Answer choice D is incorrect because a statement that a person has engaged in conduct that is substantially different from the conduct in which the person did in fact engage, such as different kinds of crime, is not considered to be true.

A man was having an affair with a woman. The man and the woman had been seen together many times in public enjoying romantic dinners. One evening, when the man claimed to be going out with friends, his wife followed him. Instead of going to meet his friends, the man met the woman at a secluded restaurant. The two sat in a dark booth, and the wife snuck in and sat unseen in the booth next to them. During their conversation, which the wife recorded using her cell phone, the woman admitted that she was pregnant with the man's child. The wife was so upset about the news that she ran out of the restaurant and forgot her cell phone. A waiter found the phone and gave it to his son, who sold used cell phones for a living. While clearing out the contents of the phone, the son discovered the recorded conversation between the man and the woman regarding her pregnancy. The son anonymously posted the recording on a popular video-sharing website. The woman's attorney eventually figured out who posted the video. Now the woman wants to sue the son for invasion of privacy. If all of the following torts are recognized in the relevant jurisdiction, which of the following causes of action would provide the woman with her best chance of recovery against the son? A Intrusion upon seclusion B Misappropriation of the right to publicity C Public disclosure of private facts D False light

Answer choice C is correct. To recover for invasion of privacy based on the public disclosure of private facts, the plaintiff must show that (i) the defendant gave publicity to a matter concerning the private life of another and (ii) the matter publicized is of a kind that would be highly offensive to a reasonable person and is not of legitimate concern to the public. Here, the woman is pregnant with the child of a married man, and the son disclosed this information to the public. This fact is not of legitimate concern to the public, and a reasonable person would find it highly offensive to have this type of information publicly disclosed. Therefore, this tort is likely to succeed. Answer choice A is incorrect. The defendant's act of intruding, physically or otherwise, into the plaintiff's private affairs, solitude, or seclusion if the intrusion is highly offensive to a reasonable person establishes liability under this cause of action. Here, the man's wife, not the son, intruded into the private affairs of the woman by recording the private conversation. The son merely published the conversation. Answer choice B is incorrect. The misappropriation of the right to publicity is the defendant's unauthorized appropriation of the plaintiff's name, likeness, or identity for the defendant's advantage without the plaintiff's consent, resulting in injury to the plaintiff. Here, there is no evidence to suggest that the son gained any type of advantage, commercial or otherwise. In fact, he posted the recording anonymously, so he is unlikely to be liable under this cause of action. Answer choice D is incorrect. In order to prove false light, the plaintiff must show that the defendant (i) made public facts about the plaintiff that (ii) placed the plaintiff in a false light, (iii) which false light would be highly offensive to a reasonable person. Most jurisdictions also require that the plaintiff prove actual malice by the defendant. Here, the son did not act with malice. Additionally, it does not appear that publication of this conversation would have suggested any false implications about the woman. Therefore, this cause of action would likely fail.

A mountain climber and his friend were scaling down a steep mountain side when one of the mountain climber's harnesses broke; although still connected to his safety rope, he dropped down the mountain a few feet. He screamed out in pain because he scraped his legs and arms badly, but when his safety rope broke his fall, he immediately saw that he was safe and in no further danger of injuring himself. Concerned that the mountain climber may be injured, the friend offered to slacken the safety rope to try to get the mountain climber down to the nearest ledge. The mountain climber adamantly refused the help because he was in no danger of being injured any further. The friend ignored the mountain climber's protestations and loosened the safety rope to drop the mountain climber down to the nearest ledge. In the process, the mountain climber sprained his ankle. The mountain climber subsequently sued the friend for battery.Can the friend be held liable for battery? A No, because the friend did not intend to cause a harmful contact to the mountain climber. B No, because the friend had implied-in-fact consent to assist the mountain climber due to an emergency. C Yes, because the friend ignored the mountain climber's protestations that he was not in danger of being injured any further. D Yes, because the mountain climber was in no actual danger.

Answer choice C is correct. A defendant is liable to the plaintiff for battery when he causes a harmful or offensive contact with the person of another, and acts with the intent to cause such contact or the apprehension of such contact. The plaintiff's consent to physical contact can be implied by silence in a situation in which a reasonable person would object to the defendant's actions, but even in an emergency situation, a competent and conscious person's right to refuse treatment must be respected. Here, the mountain climber s competently and consciously refused help or treatment from the friend, which the friend ignored. Because the friend ignored the mountain climber's protestations, he can be held liable for battery. Answer choice A is incorrect because even if the friend did not have the intent to harm the mountain climber, he exceeded the limits of acceptable contact by loosening the role despite the mountain climber's protestations. Answer choice B is incorrect. When immediate action is required to save the life or health of a person who is incapable of consenting to treatment, such consent is generally "implied in fact." However, as stated above, in this situation the mountain climber was able to refuse help from his friend. Therefore, there was no implied-in-fact consent, and the friend may be liable for battery. Answer choice D is incorrect because even if the mountain climber had been injured or in danger, he was competent and conscious enough to refuse assistance. Therefore, even real danger to the mountain climber would not have prevented the friend's liability in this situation.

An intellectually disabled man worked at a school cafeteria. In addition to serving food to the school children during the lunch shift, he also prepared the sandwiches, including sunflower butter and jelly sandwiches. One day, he ran out of sunflower butter, so he decided to make peanut butter and jelly sandwiches instead. He did not tell the cafeteria staff or the children who ordered the sunflower butter and jelly sandwiches that they were made with peanut butter. Three children who unknowingly ate the peanut butter and jelly sandwiches suffered severe allergic reactions and had to be rushed to the hospital. The parents of the children sued the man.What is the applicable standard of care? A That of a reasonable person of similar age, intelligence, and experience. B The same skill, knowledge and care as another person with similar intellectual disabilities. C That of a person with ordinary intelligence and knowledge. D That of a reasonably careful person with the same intellectual disability.

Answer choice C is correct. An intellectually disabled person is presumed to have average mental abilities and the same knowledge as an average member of the community. The defendant's own mental or emotional disability is not considered in determining whether his conduct is negligent. Answer choice A is incorrect because it states the standard of care imposed upon a child. Answer choice B is incorrect because it states the standard of care imposed upon professionals. Answer choice D is incorrect because it states the standard of care imposed upon a person with a physical disability.

As part of a fraternity dare, a college student stood in the middle of a road while drinking a beer. The driver of a car, tired of the fraternity pranks throughout the town, saw the student standing in the road, and reduced his speed but decided not to stop or swerve, saying to himself, "Well, he shouldn't be in the road anyway. He had better get out of the way, and if I hit him, it's his own fault." The intoxicated student could not get out of the way quickly enough, and the driver ran over his foot. If the student sues the driver for negligence in a contributory-negligence jurisdiction, is the driver liable for the injuries that the student sustained to his foot? A The driver would be liable only for part of the student's damages. B he driver would not be liable for the student's damages due to the student's contributory negligence. C The driver would be liable for all of the student's damages. D The driver would be liable only if he intended to cause injury to the student.

Answer choice C is correct. Even in jurisdictions that follow the common-law contributory negligence rules, such as this one, a plaintiff can mitigate his own negligence by proving that the defendant had the last clear chance to avoid injuring the plaintiff but failed to do so. Here, even though the student was negligently standing in the middle of the road, which would otherwise prevent him from recovering, the driver was actually aware of the student's peril, yet did not attempt to avoid hitting the student, even though he had the last clear chance to do so. Answer choice A is incorrect because that would be the result in a comparative-negligence jurisdiction, not a contributory-negligence jurisdiction. Answer choice B is incorrect because, while the plaintiff's own negligence would be a complete bar to recovery in a contributory-negligence jurisdiction, the plaintiff can mitigate the consequences of her own negligence by proving that the defendant had the last clear chance to avoid the injury and failed to do so. Answer choice D is incorrect because the intent to cause injury is not a required element of a negligence claim (even an intentional tort claim does not require the intent to cause injury).

A telephone company was removing wooden utility poles on a residential street and replacing them with new steel poles. The old poles were approximately 25 feet tall, and weighed several tons each. One morning, telephone company employees were removing an old pole. As a 10-year-old boy walked past the construction site, the old utility pole fell and crushed him to death. When the news was conveyed to the boy's mother, who was at work several miles away, she immediately fainted. For the next 48 hours, the mother was unable to function due to shock over the event. In the following months, the mother had difficulty sleeping due to nightmares as a result of the incident. The mother sued the telephone company for negligent infliction of emotional distress stemming from her son's death. She produced evidence at trial conclusively establishing that the telephone company was negligent in allowing the old utility pole to fall. The applicable jurisdiction has abandoned the zone of danger requirement for this type of action. Which party is likely to prevail? A The mother, because she was closely related to the boy. B The mother, because she suffered severe emotional distress. C The telephone company, because the mother was not present at the scene of the accident. D The telephone company, because its actions were not extreme and outrageous.

Answer choice C is correct. In order for a person who is not within the zone of danger to recover under a theory of negligent infliction of emotional distress, she must (i) be closely related to the person injured by the defendant, (ii) be present at the scene of the injury, and (iii) personally observe (or otherwise perceive) the injury. In the instant case, the mother was not present at the scene of the accident. Thus, the mother's claim cannot succeed. Answer choice A is incorrect because although a bystander must be closely related to the person directly harmed in order to recover, the existence of this relationship is not the sole requirement for recovery. Similarly, answer choice B is incorrect because, although the mother's fainting, shock, and sleeping problems are sufficient indicia of severe emotional distress, the existence of such distress is not the sole requirement for recovery. Answer choice D is incorrect because "extreme and outrageous conduct" is an element of intentional infliction of emotional distress, not negligent infliction of emotional distress.

A man was admitted to a hospital after complaining of persistent severe headaches. While he was there, hospital staff failed to diagnose his condition, and he was discharged. Two days later, the man died of a massive brain hemorrhage due to a congenital defect in an artery.The man's wife has brought a wrongful death action against the hospital. The wife offers expert testimony that the man would have had a "reasonable chance" (not greater than 50%) of surviving the hemorrhage if he had been given appropriate medical care at the hospital.In what type of jurisdiction would the wife's suit most likely be successful? A A jurisdiction that applies traditional common law rules concerning burden of proof. B A jurisdiction that allows recovery based on strict liability. C A jurisdiction that allows recovery for the loss of the chance of survival. D A jurisdiction that recognizes loss of spousal consortium.

Answer choice C is correct. Jurisdictions that allow recovery for the loss of the chance of survival have created an exception to the traditional common law rules for establishing cause in fact. Under the traditional rules, the wife would be required to prove that reasonable action on the part of the hospital (presumably a correct diagnosis) would, more likely than not, have led to the man's survival. Here, the wife cannot establish that the chances of the man's survival would have been greater than 50% even if he had been given appropriate medical care. A jurisdiction that allows recovery for loss of the chance of survival, however, would allow the wife to recover for the reduction in her husband's chance of surviving that was caused by the failure to properly diagnose. Answer choice A is incorrect because, as explained above, if traditional common law rules concerning burden of proof were applied, the wife would be required to prove that reasonable action on the part of the hospital (presumably a correct diagnosis) would, more likely than not, have led to the man's survival. Because the wife cannot establish that the chances of the man's survival would have been greater than 50%, the wife could not carry her burden of proof on the issue of cause in fact in such a jurisdiction. Answer choice B is incorrect because cause in fact is a necessary element of a plaintiff's case in strict liability as well as in negligence. Under either theory, the wife must establish the same thing. Answer choice D is incorrect because cause in fact is also a necessary element for loss of spousal consortium. In a loss of consortium action, the wife must establish that the hospital's negligence was the cause of her husband's death. As discussed above, the wife would not be able to do that.

In an action by a man against a pharmacy, the man offered only the following evidence:The man took a clearly written prescription to a pharmacy. The pharmacy's employee filled the prescription by providing pills with 30 milligrams of the active ingredient instead of 20 milligrams, as was prescribed. Shortly after taking the pills as directed, the man, who had no previous history of heart problems, suffered a heart attack. Overdoses of the active ingredient had previously been associated with heart problems.Does the man have a valid claim against the pharmacy? A No, because pharmacies are not strictly liable for injuries caused by incorrectly filled prescriptions. B No, because the man offered no specific proof as to the pharmacy's negligence. C Yes, because a jury could reasonably conclude that the man would not have suffered a heart attack had the pharmacy provided the correct dosage. D Yes, because by providing the 30-milligram pills rather than the 20-milligram pills, the pharmacy sold the man a defective product.

Answer choice C is correct. There is sufficient circumstantial evidence to support a conclusion that the pharmacy's employee was negligent in filling the prescription and that the consequent overdose caused the heart attack. The pharmacy would be vicariously liable for its employee's negligence under respondeat superior principles. Answer choice A is incorrect. Under some theories, pharmacies might be held strictly liable for incorrectly filled prescriptions. That fact is irrelevant here, however, because in this case, the evidence of the employee's negligence, while circumstantial, is sufficient to support a negligence claim against the pharmacy under respondeat superior principles. Answer choice B is incorrect. Although the evidence does not specify exactly how the pharmacy's employee erred, it is sufficient to support a claim of negligence against the pharmacy under respondeat superior principles. Answer choice D is incorrect. It is not necessary to establish that a product was defective in order to establish a claim in negligence. Here, there is no evidence demonstrating that the product itself was defective. If the pill bottle had been improperly labeled, the product would have been considered defective, but there is no mention of mislabeling in the facts.

A woman worked as an employee in an aviary where wild birds were bred and then sold. The owner of the aviary focused on exotic and rare birds from other countries because they sold for a higher price than domestic birds. The owner imported wild birds from countries known to have outbreaks of the bird flu, but he neglected to tell the employee because he did not believe these birds were infected with the virus. The employee did not take extra precautions, such as wearing gloves and a mask or washing her hands frequently, which caused her to have frequent contact with the birds' feathers and bodily fluids. The employee started experiencing flu-like symptoms, which then progressed to pneumonia. A series of blood tests revealed that the employee had contracted the bird flu virus.The employee sued the owner for damages and severe emotional distress; under what theory is she most likely to recover? A The owner committed a battery against her by intentionally causing her to suffer a harmful contact from birds infected with the virus. B The owner committed an assault by causing her reasonable apprehension of a harmful bodily contact from birds infected with the virus. C The owner's failure to tell the employee she was in contact with birds infected with the virus was negligent. D The owner's failure to tell the employee she was in contact with birds infected with the virus was extreme and outrageous.

Answer choice C is correct. To establish a prima facie case of negligence, a plaintiff must prove duty, breach, causation, and damages. In this case, the owner owed a duty to the employee as a reasonable person to inform her of any potential health risks associated with her job. He breached that duty by not telling her that some of the birds came from countries known to have outbreaks of the bird flu. But for the owner's failure to inform her that she was working with birds that might be infected, she would not have contracted the bird flu virus. Catching pneumonia was a foreseeable result of the owner's failure to inform the employee. Finally, the employee suffered damages. Answer choices A and B are incorrect because there is no proof of actual intent on the part of the owner to cause a harmful contact (or reasonable apprehension of a harmful contact) between the infected birds and the employee. Moreover, answer choice B is also incorrect because the employee could not be placed in reasonable apprehension of a harmful contact because she was not aware that some of the birds might be infected. Answer choice D is incorrect because although the owner should have told the employee that the birds came from countries with bird flu outbreaks, he did not intend to cause her severe emotional distress, and his failure to inform was not extreme and outrageous; it was irresponsible and negligent behavior. LEARN WHYNEXT QUESTION

A pregnant mother and a soccer coach of the mother's young child were involved in a serious verbal altercation. The argument continued escalating until the coach suddenly punched the mother in the face and pushed her to the ground. The coach immediately fled the area. The mother did not suffer serious physical injury. Nonetheless, the incident greatly traumatized her. In the weeks following the attack, she had many sleepless nights and suffered several panic attacks. The mother brought suit against the coach for intentional infliction of emotional distress. The coach filed a motion for summary judgment. How should the court rule on the motion? A Grant the motion, because the coach did not intend to cause severe emotional distress. B Grant the motion, because the mother did not suffer a significant physical injury. C Deny the motion, because a jury could find that the coach was reckless as to the risk of causing emotional distress. D Deny the motion, because the doctrine of transferred intent applies.

Answer choice C is correct. To sustain a claim of intentional infliction of emotional distress, a plaintiff must prove that she suffered severe emotional distress as a result of the defendant's conduct, that the defendant intended to cause severe emotional distress, or acted with recklessness as to the risk of causing such distress, and that the defendant's conduct was extreme and outrageous. Here, a reasonable fact finder could determine that the coach acted recklessly as to the risk of causing the mother severe emotional distress, and that his conduct was extreme and outrageous. Consequently, the court should deny the motion for summary judgment. Answer choice A is incorrect because recklessness is sufficient to satisfy the intent requirement. Answer choice B is incorrect because the physical manifestation requirement only applies to bystanders and is not implicated in this fact pattern. Answer choice D is incorrect. The traditional doctrine of transferred intent does not apply to intentional infliction of emotional distress when the defendant intended to commit a different intentional tort (such as a battery) against a different victim. However, transferred intent may apply to intentional infliction of emotional distress if, instead of harming the intended person, the defendant's extreme and outrageous conduct harms another. Here, transferred intent does not apply because the soccer coach's conduct harmed the mother, who was the intended victim.

A farmer purchased a tract of farmland adjacent to a factory. For months, the factory had emitted a foul-smelling chemical cloud that hovered approximately one foot over the farmland. The farmer invited the factory foreman to his farmland to show him the damage the chemical cloud was doing to his land. The foreman informed the farmer that there was nothing he could do, and that the previous landowner had not complained about the cloud. The farmer brought suit against the factory for trespass to his land.Will the farmer's trespass suit against the factory be successful? A No, because nothing is physically touching the farmer's property. B No, because the factory was in operation and producing the cloud before the farmer purchased the land. C Yes, because the chemical cloud constitutes a physical invasion of the property. D Yes, because the factory's agent, the foreman, physically entered the farmer's property.

Answer choice C is correct. Trespass to land occurs when a defendant's intentional conduct causes a physical invasion of the land of another. The defendant need not personally enter onto the plaintiff's land; intentionally flooding the plaintiff's land, throwing rocks onto it, or intentionally emitting particulates into the air over the land will each suffice. A trespass may be committed on, above, or below the surface of the plaintiff's land. Here, the chemical cloud produced by the factory constitutes a physical invasion of the land. Accordingly, answer choice A is incorrect. Answer choice B is incorrect because it is no defense that the previous owner acquiesced in the factory's trespass. A landowner has the right to prevent anyone from trespassing on his property unless that right has been lost through prescription or bargained or given away. Answer choice D is incorrect because, since the foreman entered the land at the farmer's invitation, the factory can assert the defense of consent.

A beekeeper lived in a cabin in the woods and kept beehives in his front yard in order to harvest honey. The cabin and hives were visible from a path that was frequently used by children as a shortcut to a nearby pond. One day, an eight-year-old child was using the path to access the pond when he noticed the beehives hanging in the front yard of the cabin. Although there was no fence surrounding the property, the beekeeper had posted warning signs in the front yard stating, "Do not trespass! Beware of the beehives!" However, the child ignored the signs and approached the nearest beehive to take a closer look. While the child was quietly looking at the beehive, his cell phone rang in his pocket. The loud noise caused the bees to swarm and sting him numerous times. Although the child survived, he suffered severe injuries and was hospitalized due to an allergic reaction.If the parents of the child sue the beekeeper for damages on the child's behalf, will they prevail? A No, because the beekeeper did not act with willful, wanton, reckless, or intentional misconduct. B No, because the child's cell phone ringing was a superseding cause. C Yes, because the beekeeper's conduct created an unreasonable risk of harm to children. D Yes, because the beekeeper is strictly liable for the child's injuries.

Answer choice C is correct. Under the "attractive nuisance" doctrine, a land possessor may be liable for injuries to children trespassing on the land if (i) an artificial condition exists in a place where the land possessor knows or has reason to know that children are likely to trespass, (ii) the land possessor knows or has reason to know that the condition poses an unreasonable risk of death or serious bodily injury to children, (iii) the children, because of their youth, do not discover or cannot appreciate the danger presented by the condition, (iv) the utility to the land possessor of maintaining the condition and the burden of eliminating the danger are slight compared to the risk of harm presented to children, and (v) the land possessor fails to exercise reasonable care to protect children from the harm. Here, the beekeeper breached this duty. The beekeeper knew that children frequently used the path near his cabin to access the pond, and that the beehives were visible from the path. Posting signs that warned trespassers to keep out because of the beehives was not enough to satisfy a reasonable duty of care in this situation. The beekeeper should have at the very least built a fence around his property or the individual trees with beehives in them. Answer choice A is incorrect. Although this is the traditional standard of care owed by landowners to trespassers, a different rule applies when the trespasser is a trespassing child. In this case, because the cabin and the front yard were visible from the path frequently used by children to access a nearby pond, it is foreseeable that they would trespass onto the beekeeper's land. Therefore, the doctrine of attractive nuisance applies. Answer choice B is incorrect. A superseding cause is any intervening event that breaks the chain of proximate causation between the defendant's tortious act and the plaintiff's harm. However, a foreseeable intervening cause will not cut off a defendant's liability. In this case, it is foreseeable that the bees could react to any loud noise and attack in a swarm. For this reason, the cell phone ringing was not a superseding cause that would cut off the beekeeper's liability. Answer choice D is incorrect. Under the Third Restatement, a wild animal is an animal that belongs to a category of animals (e.g., species) that (i) have not been generally domesticated in the United States; and (ii) are likely, unless restrained, to cause personal injury. Even if the bees counted as wild animals, a landowner is not strictly liable for injuries inflicted by any of his animals against a trespasser unless the injuries were inflicted by a vicious watchdog. Additionally, keeping bees in beehives is not an abnormally dangerous activity. Therefore, strict liability does not apply in this case. LEARN WHYNEXT QUESTION

Jumping Land is an indoor playground full of giant inflatable houses and slides upon which children can play and bounce. Large, electric air machines blow air through tubes connected to the inflatables to keep them filled. The managers of Jumping Land do a safety check of the inflatables every morning, and they also inspect the air machines to make sure that the electric motors are running correctly and that the air tubes are securely attached. The air machines are kept behind childproof gates and the owners posted signs warning that the air machines are dangerous. However, to keep them from being an eyesore, the air machines were painted in bright colors so that they fit into the festive motif of Jumping Land. A child happily jumped in the bouncy houses for a while, but eventually he got bored, and asked his father if he could take a closer look at the air machines because he enjoyed learning about how motors worked. The father acquiesced because he would be there to supervise, and unlocked the gate so his son could look at the air machine's motor. When the son got close to the air machine, the air tube suddenly disconnected and whipped the boy in the face, causing permanent damage to one of his eyes. The child's father filed a claim against Jumping Land on his son's behalf for negligence, and Jumping Land responded that it was not liable because the air machines were behind childproof gates and warning signs had been posted. Under the traditional approach regarding invitees and trespassers, is the child's father likely to be successful in the suit against Jumping Land? A Yes, because the air machines used to fill the inflatables were abnormally dangerous. B Yes, because the air machines painted in bright colors were an attractive nuisance. C No, because the child went into an area from which he was prohibited to enter. D No, because the air machine was bounded by a childproof gate and warning signs.

Answer choice D is correct. A land possessor owes an invitee the duty of reasonable care, including the duty to use reasonable care to inspect the property, discover unreasonably dangerous conditions, and protect the invitee from them. Here, the child was an invitee, and Jumping Land owed him a duty of reasonable care to inspect the facility and protect the child from unreasonably dangerous conditions. Jumping Land satisfied this duty as evidenced by the daily safety inspections. However, in this case, the child was a trespasser who exceeded the scope of the invitation at the time of the injury because his father opened the childproof gate for him. The duty of reasonable care does not extend to trespassers. However, because the brightly painted air machines would likely qualify as an attractive nuisance, Jumping Land had a duty to exercise reasonable care in protecting the child from unreasonably dangerous conditions. It satisfied this duty as evidenced by the childproof gates and warning signs surrounding the air machines. Answer choice A is incorrect because the air machines are not abnormally dangerous. Abnormally dangerous means that an activity creates a foreseeable and highly significant risk of physical harm even when reasonable care is exercised, and the activity is not commonly engaged in. In this case, the use of the air machines does not create a foreseeable and highly significant risk of physical harm even when reasonable care is exercised. By performing daily inspections and gating the air machines, as well as posting warning signs, Jumping Land exercised reasonable care which minimized any foreseeable risk of physical harm. Thus, the air machines do not fit the definition of abnormally dangerous. Answer choice B is incorrect because although the brightly painted air machines were an attractive nuisance, Jumping Land exercised reasonable care in protecting the children from harm. Answer choice C is incorrect because the brightly painted air machines were an attractive nuisance, and therefore Jumping Land still owed the child, a trespasser, a duty of reasonable care.

After his car broke down on the side of a country road, a driver wandered onto an adjacent property to seek the use of a telephone. Unbeknownst to the driver, the property owner bred large dogs. The particular breed has a reputation for being dangerous, and members of the breed are typically used as watchdogs. The specific dogs owned by this breeder, however, are known among nearby residents as being extremely friendly. The driver climbed over a large wooden fence to reach the property. Once the dogs spotted the driver, three of the largest dogs ran to greet him, though he assumed the dogs planned to attack. Excessively frightened, the driver attempted to jump over a nearby barbed-wire fence and severely cut his legs. When the dogs finally reached the man, they did nothing more than lick his face until their owner arrived. The driver sued the owner under a strict liability theory for the damages resulting from his injury.Is the driver likely to succeed in his suit? A Yes, because the injury was caused by the driver's reaction to a dangerous propensity that is characteristic of the breed of dog. B Yes, because landowners are strictly liable for injuries inflicted by their animals. C No, because undiscovered trespassers cannot bring strict liability suits against landowners. D No, because the dogs were not vicious watchdogs.

Answer choice D is correct. A landowner is not strictly liable for injuries inflicted by his animals against a trespasser, except for injuries inflicted by a vicious watchdog. As stated in these facts, these dogs are not vicious and did not actually attack the driver, and the landowner would not be strictly liable for the injuries. It is irrelevant that this breed of dog is typically used as a watchdog and that the driver believed the dogs were vicious. Answer choice A is incorrect because this standard of liability is only applicable to owners of wild animals; owners of wild animals may be liable for injuries resulting from a fearful reaction to those animals. Because dogs are generally domesticated in the United States, they are not considered wild animals. Answer choice B is incorrect because landowners are not strictly liable for injuries inflicted by their animals unless the animals are vicious watchdogs, wild animals, or animals known by the animal's owner to have abnormally dangerous propensities and the harm results from those dangerous propensities. Answer choice C is incorrect because landowners may be strictly liable to trespassers for injuries inflicted by certain animals.

A man lived in a politically competitive district within a swing state, and was therefore constantly approached by political canvassers. The man was annoyed with never-ending knocks on his door, and put up a "No Soliciting" sign. One day, a canvasser knocked on the man's door, despite the sign, and when the man yelled through the door at the canvasser to read the sign, the canvasser rang the doorbell. The man was not afraid, but was infuriated that the canvasser had the audacity to ring his doorbell despite the sign. Instead of asking him to leave, the man grabbed a stun gun, opened the front door, and immediately discharged it at the canvasser, intending to frighten him off the property. The canvasser had a preexisting heart condition and died from cardiac arrest triggered by the stun gun. The executor of the canvasser's estate sued the man for wrongful death.If the court finds that the use of the stun gun constituted deadly force, is the executor likely to prevail? A No, because deadly force can be used to defend property if one's personal safety is threatened. B No, because the man did not set a trap to defend his property from trespassers. C Yes, because the man did not ask the canvasser to leave his property before shooting him. D Yes, because the man did not believe his personal safety was threatened by the canvasser.

Answer choice D is correct. A person may use reasonable force to defend his property if he reasonably believes it is necessary to prevent tortious harm to his property. However, deadly force may not be used in defense of property unless one's personal safety is threatened. Based upon the facts, there is no evidence that the man was in fear for his life. He only wanted the man to leave his property. Thus, the use of deadly force was not appropriate in this instance. Answer choice A is incorrect because the facts provide that the man did not fear for his own safety. Answer choice B is incorrect. Although the use of a spring-gun or other trap set to expose a trespasser to a force likely to inflict death or grievous bodily injury will lead to liability for the land possessor, this is not the only way that the man could have been liable for an injury to a trespasser. Here, the man is liable because, regardless of his duties as a landowner, he used deadly force to protect his property when he was not in fear for his physical safety. Answer choice C is incorrect. In general, a landowner is entitled to use reasonable force only after making a request that the trespasser desist and the trespasser ignores the request. However, because such a request is not required if the landowner reasonably believes that a request will be useless or that substantial harm will be done before it can be made, the man would have been permitted to use reasonable force without asking the canvasser to leave if he feared for his personal safety. However, even then, the use of force may only be reasonable, and deadly force was not reasonable in this case.

A journalist received information from a credible source that a large corporation had used child labor in foreign countries to produce electronics. The corporation had previously criticized its main competitor for using unethical labor practices in foreign countries. The journalist published an article in a nationally distributed newspaper repeating the charges against the corporation, which resulted in boycotts of the corporation's products. The corporation sued the journalist for defamation and produced records showing that no child labor was ever used in the production of its products. The journalist's source testified at trial that she knew the statements she made were false. The journalist testified that the article was based solely on the source's statements, and that no other research was conducted into the veracity of her statements. He further testified that he believed the source, and never doubted the truthfulness of her information. The trier of fact found the testimony of the source and the journalist to be credible.Is the corporation likely to prevail in its defamation suit against the journalist? A Yes, because the journalist repeated the source's defamatory statement. B Yes, because the source acted with malice in making the statement. C No, because only an individual, and not a corporation, may maintain a defamation action. D No, because the journalist did not act with malice.

Answer choice D is correct. A plaintiff may bring an action for defamation if the defendant's defamatory language is of or concerning the plaintiff, is published to a third party who understands its defamatory nature, and damages the plaintiff's reputation. For matters of public concern, the plaintiff must prove fault on the part of the defendant as well as the falsity of the statement. The standard for establishing the defendant's fault depends on the plaintiff's status as a public or private figure. A public figure is someone who is known to the general public and includes any person who has voluntarily injected himself into the public eye. If the statement relates to a public figure, the plaintiff must prove actual malice on the part of the defendant, as well as the falsity of the language. Malice means that the defendant knew that the statement or language was false, or he recklessly disregarded its truthfulness. To establish a reckless disregard for the truthfulness of a statement, the defendant must have entertained serious doubts about its truthfulness; mere failure to check facts is not sufficient. In this case, the corporation was a public figure, and thus would need to prove malice. Because the journalist testified credibly that he did not doubt the truthfulness of the statement, the corporation is unlikely to succeed in demonstrating malice, and thus would not prevail in its defamation claim. Answer choice A is incorrect because the corporation cannot show that the journalist acted with malice in repeating the statement. Answer choice B is incorrect because the corporation must show that the journalist, and not just the source, made the statement with malice. Answer choice C is incorrect because corporations and groups may maintain defamation actions.

In December, a contractor was hired by a power utility company to perform repair work on a large transformer. The contractor performed the work negligently and as a result severely damaged one of the conducting coils in the transformer. The damage resulted in a two-day power outage in a town with a large industrial park. An electronics manufacturer was a tenant in the industrial park, and the power outage crippled its ability to meet the strong demand for its products during the critical holiday buying season. While none of the electronic manufacturer's machines were damaged, it can prove with certainty that the power outage directly caused it to lose $750,000 in business. The electronics manufacturer sued the power utility company and the contractor for negligently causing its sales losses. If, at the end of the plaintiff's case, both defendants move for summary judgment, and all the foregoing facts are undisputed, how should the court rule on the motions? A Deny both motions, because both parties were substantial factors in the electronics manufacturer's loss. B Deny both motions, because the burden of proof has shifted to both defendants to exonerate themselves. C Grant the motion as to the contractor, but deny the motion as to the power utility company, because liability is assigned to the principal under the respondeat superior doctrine. D Grant both motions, because the electronics manufacturer suffered no tangible injury to its equipment or employees.

Answer choice D is correct. A plaintiff who suffers only economic loss without any related personal injury or property damage cannot recover such loss through a negligence action. Here, the electronics manufacturer's $750,000 loss is purely economic, so any negligence claim is improper. For this reason, answer choices A, B, and C are all incorrect.

The guest of a member of a condominium association was injured when he slipped on a wet spot in the front lobby area. This area is owned by each unit owner as a tenant-in-common with the other unit owners. The guest sued the association, and a jury determined that the association was liable for injury to the member's guest. The jury awarded the guest damages.Which of the following best describes the responsibility of each member for the damages? A No member bears any financial responsibility for the damages. B Each member is jointly liable for the total amount of the damages. C Each member is jointly and severally liable for the total amount of the damages. D Each member is liable for her share of the association expenses, which includes payment of the judgment awarded to a tort plaintiff.

Answer choice D is correct. Although each member of a condominium association owns the common areas of the condominium as a tenant-in-common with the other members, the member is only indirectly responsible for a tort judgment against the association. Payment of the tort judgment by the association is an expense of the association, and each member is liable for a share of the association's expenses. For this reason, answer choices B and C are incorrect. Answer choice A is incorrect because while a member is not directly responsible for payment of a tort judgment awarded against the association, the member does bear a portion of the financial responsibility for the judgment as an association expense. Consequently, answer choice D is the better alternative. LEARN WHYNEXT QUESTION

A pest control company fumigated one of two buildings in an apartment complex with a toxic gas in order to eliminate unwanted insects. Even though the company exercised reasonable care, the gas escaped into the other building, which adjoined the fumigated building, where the gas caused serious illness to a tenant in that building. The tenant had received a written advance notice about the fumigation that advised the tenant of the need to vacate his apartment during the hours the fumigation was conducted. The tenant chose instead to remain there in order to watch a favorite television program. The applicable jurisdiction treats fumigation as an ultrahazardous activity. The injured tenant filed an action against the pest control company. Who will prevail? A The tenant, because the pest control company is strictly liable for the harm that resulted from the fumigation. B The tenant, because the pest control company was negligent in conducting the fumigation. C The pest control company, because the tenant was not a resident of the fumigated building. D The pest control company, because the tenant assumed the risk.

Answer choice D is correct. Although the pest control company, by engaging in an abnormally dangerous activity, is strictly liable for harm that results from the conduct of that activity, assumption of the risk is a defense to strict liability. The tenant's decision to remain in the apartment and thereby possibly expose himself to the gas was both knowing and voluntary. Answer choice A is incorrect because, as noted, even though the company is strictly liable, assumption of the risk is a defense to strict liability. Answer choice B is incorrect because the company exercised reasonable care in fumigating the building. Answer choice C is incorrect because the company's strict liability to the tenant is not based on residency in the fumigated building.

A police officer was on patrol in his police car when he noticed a pickup truck that appeared to be dangerously overloaded with bales of hay, speeding on an expressway. The policeman drove behind the pickup truck to get a closer look. When the driver of the pickup truck saw the police car behind him, the driver slowed down and swerved abruptly from the fast lane into the slow lane. The sudden swerve caused the load of hay to shift and a bale to fall out. The bale of hay struck the window of the police officer's car, causing him to lose control of the car and crash into a sign post. The policeman suffered significant injuries in the crash.Should the police officer be able to recover from the driver for his injuries? A Yes, but only if the driver was actually speeding. B Yes, but only if the driver had been negligent in loading the pickup truck. C Yes, but only if the policeman was not comparatively negligent in following the pickup truck. D No.

Answer choice D is correct. An emergency professional, such as a police officer or firefighter, is barred from recovering damages from the party whose negligence caused the professional's injury if the injury results from a risk inherent in the job ("firefighter's rule"). Here, the police officer's injury resulted from a risk inherent in the job. The driver's abrupt swerving of his truck in response to the police officer's presence was a risk involved in a police pursuit/investigation. Therefore, the injuries sustained by the police officer in the crash as a result of the driver's actions were inherent to his job and, consequently, the police officer cannot recover from the driver for his injuries. Answer choice A is incorrect because the rule bars recovery even if the driver was negligent. Answer choice B is incorrect for the same reason. Answer choice C is incorrect because the rule precludes recovery even when the plaintiff was not comparatively negligent.

A customer pledged a stock certificate to a bank as security for a loan. A year later, when the customer fully repaid the loan, the bank refused the customer's demand to return the stock certificate because the officer dealing with the loan had the mistaken belief that there was still a balance due. No one at the bank reviewed the records until two months later, at which time the error was discovered. The bank then offered to return the stock certificate. However, the customer refused to accept it.At the time the customer pledged the certificate, the shares were worth $10,000; at the time the customer repaid the loan, the shares were worth $20,000; and at the time the bank offered to return the certificate, the shares were worth $5,000.If the customer brings an action against the bank based on conversion, how much, if anything, should the customer recover? A Nothing, because the bank lawfully came into possession of the certificate. B $5,000, because that was the value of the shares when the customer refused to accept the certificate back. C $10,000, because that was the value of the shares when the bank came into possession of the certificate. D $20,000, because that was the value of the shares when the customer was entitled to the return of the certificate.

Answer choice D is correct. Conversion is equivalent to a forced sale of the chattel to the defendant, who is liable for the full value of the chattel at the time of the tort. The tort occurred when the bank refused to relinquish the stock certificate in response to the customer's lawful demand, and at that time the shares were worth $20,000. Answer choice A is incorrect. Someone who refuses to surrender a chattel to another person who is entitled to its immediate possession is liable for conversion even if the one holding the chattel originally came into possession lawfully. The bank is liable to the customer for the value of the shares at the time the bank refused the customer's lawful demand for return of the certificate - $20,000. Answer choices B and C are incorrect because the defendant is liable for the full value of the chattel at the time of the tort - when the bank refused to relinquish the stock certificate in response to the customer's lawful demand, and at that time the shares were worth $20,000.

An adult woman was vacationing at a friend's house on a lake. One afternoon, the woman watched her friend maneuver his motorized personal watercraft around the lake; the friend took a particularly violent spill that temporarily knocked the wind out of him but left him otherwise unharmed. The next morning, without the friend's knowledge, she decided to take the personal watercraft out on the lake herself. Due to her inability to control the vehicle, it flipped over. As a consequence, the woman suffered serious physical injuries. The woman brought a lawsuit against the friend to recover damages for her injuries. The applicable jurisdiction has adopted comparative negligence rules. Prior to the submission of the case to the jury, the friend requested that the court specifically instruct the jury on the assumption of the risk defense. Should the court grant this request? A Yes, because the woman voluntarily assumed the risk of being injured. B Yes, because assumption of the risk is an absolute bar to recovery. C No, because the defendant did not have the requisite knowledge for this defense. D No, because assumption of the risk is not recognized as a separate defense.

Answer choice D is correct. In a comparative negligence jurisdiction, assumption of the risk is not recognized as a separate defense—it has been merged into the comparative-fault analysis and merely reduces recovery. The plaintiff's awareness of the risk of her conduct is generally taken into account in determining the degree to which she is at fault, but it can also be considered in determining the reasonableness of the plaintiff's or the defendant's actions. Answer choice A is incorrect because, even assuming that the woman did voluntarily assume that risk, such a defense is not recognized as a separate defense in a comparative negligence jurisdiction. Answer choice B is incorrect because assumption of the risk is only an absolute bar to recovery in a contributory negligence jurisdiction, not a comparative negligence jurisdiction. Answer choice C is incorrect because assumption of the risk requires the plaintiff, not the defendant, to be aware of the risks of the plaintiff's conduct.

A skier was riding on a ski lift at a large resort. Passengers on the lift were seated on chairs, and were secured onto the chairs with lap bars. While riding on the lift, the skier's cell phone rang in her pocket, and she momentarily removed her lap bar to reach the phone. While the lap bar was unfastened, the ski lift suddenly malfunctioned, jerking the skier forward and out of her seat. She fell 15 feet to the ground and, as a result, suffered a broken leg. Several other passengers on the lift were also seriously injured as a result of the accident.The ski lift was manufactured, maintained, and operated by a large ski lift corporation. The skier sued the ski lift corporation in a jurisdiction that applies contributory negligence rules. At trial, the skier could produce no direct evidence of negligence on the part of the ski lift corporation, but claimed that her injury could not have occurred without their negligence.Which of the following questions will not be at issue in this negligence action? A Would the skier's injury have occurred if she had not removed the lap bar? B Did any outside parties have access to the ski lift machinery? C Would the skier's injury have occurred if the ski lift corporation exercised reasonable care? D Were the skier's actions the primary cause of the accident?

Answer choice D is correct. The jurisdiction in which the lawsuit is proceeding applies the contributory negligence rule, which bars recovery if a plaintiff is at all responsible for her injury. Consequently, the question is not whether the skier's own negligence was the primary cause of the accident (which would be a question in a jurisdiction that bars recovery if a plaintiff is more than 50% responsible for her injury), but rather, whether the plaintiff was at all negligent. For that reason, answer choice A is incorrect. Answer choices B and C are incorrect because they articulate two of the factors implicated in a res ipsa loquitur case, which would apply under these facts.

A borrower owed a substantial sum of money to an unsavory lender. One afternoon, the lender knocked on the borrower's door. When the borrower opened the door, the lender was holding a baseball bat and said, "If you don't get me the money you owe within the next two hours, I'll break your legs." The borrower was extremely frightened, and immediately gave the lender the cash needed to satisfy the debt. If the borrower later sues the lender for assault, will the borrower prevail? A Yes, because the lender threatened the borrower with harmful or offensive bodily contact. B Yes, because the lender intended to place the borrower in apprehension of harmful or offensive bodily contact. C No, because the lender's words alone cannot give rise to an assault claim. D No, because the lender gave the borrower two hours to deliver the money.

Answer choice D is correct. To recover for assault, a plaintiff must prove that the defendant's intentional action or threat caused the plaintiff to experience reasonable apprehension of an imminent harmful or offensive bodily contact. Here, the lender threatened the borrower with harm two hours later in time; thus, the threatened harmful bodily contact was not imminent, and an assault claim cannot prevail. Answer choice A is incorrect because although the lender threatened the borrower with harmful or offensive bodily contact, the threat was not imminent. Answer choice B is incorrect because while the lender intended to place the borrower in apprehension of harmful or offensive bodily contact, the requisite intent for assault, the lender's words and conduct did not create the apprehension of an imminent contact. Answer choice C is incorrect because, while words alone generally do not constitute an assault, they may be sufficient if coupled with conduct or other circumstances. The lender's words along with his conduct (i.e., carrying a baseball bat) are sufficient to constitute an assault.

A housecleaner purchased a textile iron manufactured by an appliance manufacturer. The housecleaner successfully used the iron for a few weeks, ironing clothing for her clients. One day, the housecleaner accidentally knocked an expensive oil painting into her mop bucket, causing the painting to get wet. The housecleaner let the painting dry, but the canvas was left wrinkled and puckered. The housecleaner tried to use the iron on its lowest setting to iron out the wrinkles. When she did, the iron sparked, the oil painting caught on fire, and the housecleaner suffered burns on her hands and arms. The housecleaner sued the appliance manufacturer under a strict products liability theory of recovery, alleging a design defect. At trial, it was established that the wiring in the iron had sparked due to the cheap insulation material used by the manufacturer, and that the manufacturer could have prevented this by using a safer type of insulation at the same cost. However, the manufacturer established that the cheap insulation it used complied with governmental safety standards. The relevant jurisdiction is a traditional contributory-negligence jurisdiction and applies the risk-utility test to determine whether a design defect exists.Will the housecleaner succeed in her suit against the manufacturer? A Yes, but her recovery will be reduced due to her misuse of the iron. B Yes, because the defect regarding the safety mechanism existed when the iron left the manufacturer's control. C No, because the manufacturer complied with governmental safety standards. D No, because the iron was not used by the house cleaner in a foreseeable way.

Answer choice D is correct. To recover under a strict products liability theory of recovery, the plaintiff must prove that the product was defective, the defect existed at the time the product left the defendant's control, and the defect caused the plaintiff's injuries when the product was used in an intended or reasonably foreseeable way. Here, the iron was defective, and a design defect existed when it left the manufacturer's control. However, the housecleaner did not use the iron in an intended or reasonably foreseeable way. The misuse, alteration, or modification of a product by the user in a manner that is neither intended by nor reasonably foreseeable to the manufacturer typically negates liability. The textile iron was not intended to be used to dry something as combustible as an oil painting, and this misuse of the iron will probably prevent the housecleaner from recovering. Answer choice A is incorrect because most contributory-negligence jurisdictions hold that product misuse totally bars recovery. Answer choice B is incorrect because, although the defect in the iron existed when it left the manufacturer's control, the housecleaner's unforeseeable misuse of the iron will likely bar her recovery. Answer choice C is incorrect because compliance with governmental safety standards is not conclusive evidence that the product is not defective. Therefore, answer choice D is the best answer. LEARN WHYNEXT QUESTION

A zookeeper adopted an injured orangutan from the zoo. Many years of living in the zoo had tamed the orangutan, and he had no known dangerous propensities, so the zookeeper allowed the orangutan to live inside his home. The orangutan was very intelligent, and had learned how to open doors and mimic other basic human behaviors. One day, while the zookeeper was in the shower, a salesman selling home insurance rang the doorbell. The orangutan heard the doorbell ring and opened the door. In fear for his life when he saw the orangutan open the door, the salesman collapsed from a non-lethal heart attack. The salesman filed a strict liability action against the zookeeper to recover damages. The jurisdiction applies the common-law rules for contributory negligence and assumption of the risk. Is the salesman likely to succeed in his action against the zookeeper? A No, because the orangutan had no known dangerous propensities. B No, because the orangutan did not directly cause the salesman's injuries. C Yes, because the zookeeper failed to use reasonable care to restrain the orangutan. D Yes, because the zookeeper is liable for the salesman's fearful reaction to the orangutan.

Answer choice D is correct. Under the Third Restatement, a wild animal is an animal that belongs to a category of animals (e.g., species) that have not been generally domesticated in the United States and are likely, unless restrained, to cause personal injury. (The Second Restatement's definition is broader, not limited to animals that might cause injury.) Strict liability applies to an injury caused by a plaintiff's fearful reaction to the sight of an unrestrained wild animal, in addition to injuries caused directly by the wild animal. In this case, although the orangutan was tamed, he still qualifies as a wild animal, and the zookeeper is strictly liable for injuries caused by the salesman's fearful reaction to the orangutan. Answer choice A is incorrect because there is no requirement for an individual wild animal to have known dangerous propensities for strict liability to apply. An individual wild animal remains a wild animal based on the propensities of its species as a whole, even when this particular animal has been tamed for a number of years and even though its departure from that tameness is sudden and unexpected. Answer choice B is incorrect because, as stated above, the zookeeper is strictly liable for any injuries caused by the salesman's fearful reaction when he saw the orangutan open the door. The orangutan did not need to directly cause the salesman's injuries for the zookeeper to be liable. Answer choice C is incorrect because the zookeeper would be strictly liable for any injuries resulting from the salesman's fearful reaction to the orangutan, regardless of the care taken to restrain the wild animal, if the orangutan had still been unrestrained when the salesman encountered him.

The driver of a car and a motorcyclist collided at an intersection. Debris from the accident struck and injured a nearby pedestrian. The pedestrian sued both drivers for his injury. It was determined that the motorcyclist was 75% responsible for the injury and the car driver 25%. The driver of the car filed a motion to dismiss the claim against him.The applicable jurisdiction has adopted pure several liability.How should the court rule on the driver's motion? A Grant the motion, because the motorcyclist is liable for the full amount of damages. B Grant the motion, because the pedestrian may not collect damages from both parties. C Deny the motion, because the pedestrian may sue the driver of the car for the entire amount of her damages. D Deny the motion, because the plaintiff may sue the driver of the car for 25% of the damages she suffered.

Answer choice D is correct. Unlike in a joint and several liability jurisdiction, in which each tortfeasor who causes a single indivisible harm is jointly and severally liable for the entire amount of damages suffered by the plaintiff, in a pure several liability jurisdiction, such a tortfeasor is only liable for that tortfeasor's comparative share of the plaintiff's damages. Accordingly, since the jurisdiction has adopted pure several liability, the pedestrian may sue the car driver for the portion of the pedestrian's damages attributable to the driver's fault. Answer choice A is incorrect because, even though the motorcyclist was more at fault than the car driver, each is liable for the proportion of damages that he caused. Answer choice B is incorrect because the pedestrian may collect from each defendant the proportion of total damages that that defendant caused. Answer choice C is incorrect because the pedestrian cannot sue the car driver for the entire amount of her damages since the jurisdiction follows the pure several liability rule.

A consumer purchased a ladder from a hardware store for use around the house. Due to a defect in the design of the ladder, the consumer fell from the ladder and was seriously injured. The manufacturer of the ladder had affixed a notice to the ladder that limited consequential damages from any defect in the ladder or from a breach of the implied warranty of merchantability. The consumer had read the notice prior to purchasing the ladder. The consumer brought an action based on both a products liability claim and breach of the implied warranty of merchantability claim against the manufacturer to recover damages for his personal injuries. Can either claim support the consumer's recovery? A No, as to either type of claim. B Yes, as to the products liability claim, but no, as to the implied warranty of merchantability claim. C Yes, as to the implied warranty of merchantability claim, but no, as to the products liability claim. D Yes, as to both types of claims.

Answer choice D is correct. With respect to a products liability claim, a limitation of consequential damages by a product seller or other distributor does not generally bar or reduce an otherwise valid products liability claim for personal injury. With respect to a breach of the implied warranty of merchantability, while a limitation of consequential damages is permitted, the limitation of such damages for personal injury in the case of consumer goods is prima facie unconscionable. Consequently, answer choices A, B, and C are incorrect.

WHAT'S A contributory negligence jurisdiction?

In a contributory negligence jurisdiction, the plaintiff's contributory negligence is a complete bar to the plaintiff's recovery.

pure comparative negligence jurisdiction?

In a pure comparative negligence jurisdiction (remember that you should always apply pure comparative negligence on the MBE unless you are told to do otherwise), the failure to mitigate is taken into account, but it does not categorically prevent recovery for such harm. Rather, the failure to mitigate will reduce the amount of damages that may be recovered

Strict products liability elements

In order to recover under a strict products liability theory, a plaintiff must prove that the product was defective (in manufacture, design, or failure to warn), the defect existed when it left the defendant's control, and the defect caused the plaintiff's injuries when the product was used in an intended or reasonably foreseeable way. The misuse, alteration, or modification of a product by the user in a manner that is neither intended by nor reasonably foreseeable to the manufacturer typically negates liability.

"attractive nuisance" doctrine

Under the "attractive nuisance" doctrine, a land possessor may be liable for injuries to children trespassing on the land if: (i) an artificial condition exists in a place where the land possessor knows or has reason to know that children are likely to trespass, (ii) the land possessor knows or has reason to know that the condition poses an unreasonable risk of death or serious bodily injury to children, (iii) the children, because of their youth, do not discover or cannot appreciate the danger presented by the condition, (iv) the utility to the land possessor of maintaining the condition and the burden of eliminating the danger are slight compared to the risk of harm presented to children, and (v) the land possessor fails to exercise reasonable care to protect children from the harm.

doctrine of res ipsa loquitur?

Under the doctrine of res ipsa loquitur, negligence may be inferred if (1) the injury is the type that ordinarily would not occur without negligence, (2) the instrumentality causing the injury was within the exclusive control of the defendant, and (3) the plaintiff was not responsible for his injury.

res ipsa loquitur (as proof of breach)

Under the doctrine of res ipsa loquitur, the trier of fact may infer the existence of the defendant's negligent conduct in the absence of direct evidence of negligence. The plaintiff must prove that: (1) the accident is of a kind which ordinarily does not occur in the absence of negligence; (2) the harm was caused by an agent or instrumentality within the exclusive control of the defendant; and (3) the harm was not due to any action on the part of the plaintiff.


Kaugnay na mga set ng pag-aaral

CHAPTER 46 Management of Patients with Oral and Esophageal Disorders QUIZ

View Set

Taxes,Retirement,and Other Insurance Concepts

View Set

Chp 28 Regulation of Gene Expression

View Set

Square Roots & Estimating Square Roots, Solving Radical Equations LD, Radical Equations, Fractions & Exponents Math Skills, Fractional Exponents, Fractional Exponents

View Set